Fixed point of a function defined in terms of a metric on a compact metric space












2












$begingroup$


Let $X$ be a compact metric space, and assume $f : X → X$ satisfies
$$d(f(x), f(y))< d(x, y), forall x neq y ∈ X.$$
Define a function $g : X → mathbb{R}$ by $g(x) = d(x,f(x)), forall x ∈ X$. Prove the following statements.



a) $|g(x)-g(y)| leq 2d(x,y), forall x,y in X$.



b) $g$ is continuous and $g(x)=0$ for some $x in X$.



$text{Proof:}$



a) Observe that
begin{align*}|g(x)-g(y)|
&=|d(x,f(x))-d(y,f(y))| \
&=|d(x,f(x))-d(f(x),y)+d(f(x),y)-d(y,f(y))| \
&leq |d(x,f(x)-d(f(x),y)|+|d(f(x),y)-d(y,f(y))| \
&leq d(x,y)+d(f(x),f(y)) \
&leq 2d(x,y),
end{align*}

whre in the last step we used the hypothesis that $d(f(x), f(y))< d(x, y), forall x neq y ∈ X.$



b) To show continuity we take $delta= largefrac{varepsilon}{2}$. Then if $d(x,y)< delta$, we have by part a) that $|g(x)-g(y)|< varepsilon$.



To show that there exists $xin X$ such that $g(x)=0$, we follow AlexL's answer and note that since $g$ is continuous and $X$ is compact, $g$ attains its minimum at some $x in X$. Suppose that for that for $g(x)neq 0$, and so $f(x)neq x$. Then
$$g(f(x))=d(f(x),f(f(x)))<d(x,f(x))=g(x),$$
which is a contraction to $g$ attaining its minimum at $x$. It follows that $g(x)=0$, and this completes the proof.










share|cite|improve this question











$endgroup$








  • 1




    $begingroup$
    You have a small error. In the statement of a) an in the last displayed line of he proof of a) the inequality $<$ should be $leq$ to include the case $x=y.$ It would be easier to make this change than to re-write it to cover only the case $xne y$.
    $endgroup$
    – DanielWainfleet
    Nov 1 '18 at 7:47






  • 1




    $begingroup$
    Another way to prove that $g$ is continuous: It suffices that if $ lim_{nto infty}d(x,y_n)=0$ then $lim_{nto infty}|g(x)-g(y_n)|=0.$ Use your proof of a), writing $y_n$ for $y.$ Then $|g(x)-g(y_n)| leq 2d(x,y_n)$..... So if $d(x,y_n)to 0$ then $|g(x)-g(y_n)|to 0 .$
    $endgroup$
    – DanielWainfleet
    Nov 1 '18 at 7:59


















2












$begingroup$


Let $X$ be a compact metric space, and assume $f : X → X$ satisfies
$$d(f(x), f(y))< d(x, y), forall x neq y ∈ X.$$
Define a function $g : X → mathbb{R}$ by $g(x) = d(x,f(x)), forall x ∈ X$. Prove the following statements.



a) $|g(x)-g(y)| leq 2d(x,y), forall x,y in X$.



b) $g$ is continuous and $g(x)=0$ for some $x in X$.



$text{Proof:}$



a) Observe that
begin{align*}|g(x)-g(y)|
&=|d(x,f(x))-d(y,f(y))| \
&=|d(x,f(x))-d(f(x),y)+d(f(x),y)-d(y,f(y))| \
&leq |d(x,f(x)-d(f(x),y)|+|d(f(x),y)-d(y,f(y))| \
&leq d(x,y)+d(f(x),f(y)) \
&leq 2d(x,y),
end{align*}

whre in the last step we used the hypothesis that $d(f(x), f(y))< d(x, y), forall x neq y ∈ X.$



b) To show continuity we take $delta= largefrac{varepsilon}{2}$. Then if $d(x,y)< delta$, we have by part a) that $|g(x)-g(y)|< varepsilon$.



To show that there exists $xin X$ such that $g(x)=0$, we follow AlexL's answer and note that since $g$ is continuous and $X$ is compact, $g$ attains its minimum at some $x in X$. Suppose that for that for $g(x)neq 0$, and so $f(x)neq x$. Then
$$g(f(x))=d(f(x),f(f(x)))<d(x,f(x))=g(x),$$
which is a contraction to $g$ attaining its minimum at $x$. It follows that $g(x)=0$, and this completes the proof.










share|cite|improve this question











$endgroup$








  • 1




    $begingroup$
    You have a small error. In the statement of a) an in the last displayed line of he proof of a) the inequality $<$ should be $leq$ to include the case $x=y.$ It would be easier to make this change than to re-write it to cover only the case $xne y$.
    $endgroup$
    – DanielWainfleet
    Nov 1 '18 at 7:47






  • 1




    $begingroup$
    Another way to prove that $g$ is continuous: It suffices that if $ lim_{nto infty}d(x,y_n)=0$ then $lim_{nto infty}|g(x)-g(y_n)|=0.$ Use your proof of a), writing $y_n$ for $y.$ Then $|g(x)-g(y_n)| leq 2d(x,y_n)$..... So if $d(x,y_n)to 0$ then $|g(x)-g(y_n)|to 0 .$
    $endgroup$
    – DanielWainfleet
    Nov 1 '18 at 7:59
















2












2








2





$begingroup$


Let $X$ be a compact metric space, and assume $f : X → X$ satisfies
$$d(f(x), f(y))< d(x, y), forall x neq y ∈ X.$$
Define a function $g : X → mathbb{R}$ by $g(x) = d(x,f(x)), forall x ∈ X$. Prove the following statements.



a) $|g(x)-g(y)| leq 2d(x,y), forall x,y in X$.



b) $g$ is continuous and $g(x)=0$ for some $x in X$.



$text{Proof:}$



a) Observe that
begin{align*}|g(x)-g(y)|
&=|d(x,f(x))-d(y,f(y))| \
&=|d(x,f(x))-d(f(x),y)+d(f(x),y)-d(y,f(y))| \
&leq |d(x,f(x)-d(f(x),y)|+|d(f(x),y)-d(y,f(y))| \
&leq d(x,y)+d(f(x),f(y)) \
&leq 2d(x,y),
end{align*}

whre in the last step we used the hypothesis that $d(f(x), f(y))< d(x, y), forall x neq y ∈ X.$



b) To show continuity we take $delta= largefrac{varepsilon}{2}$. Then if $d(x,y)< delta$, we have by part a) that $|g(x)-g(y)|< varepsilon$.



To show that there exists $xin X$ such that $g(x)=0$, we follow AlexL's answer and note that since $g$ is continuous and $X$ is compact, $g$ attains its minimum at some $x in X$. Suppose that for that for $g(x)neq 0$, and so $f(x)neq x$. Then
$$g(f(x))=d(f(x),f(f(x)))<d(x,f(x))=g(x),$$
which is a contraction to $g$ attaining its minimum at $x$. It follows that $g(x)=0$, and this completes the proof.










share|cite|improve this question











$endgroup$




Let $X$ be a compact metric space, and assume $f : X → X$ satisfies
$$d(f(x), f(y))< d(x, y), forall x neq y ∈ X.$$
Define a function $g : X → mathbb{R}$ by $g(x) = d(x,f(x)), forall x ∈ X$. Prove the following statements.



a) $|g(x)-g(y)| leq 2d(x,y), forall x,y in X$.



b) $g$ is continuous and $g(x)=0$ for some $x in X$.



$text{Proof:}$



a) Observe that
begin{align*}|g(x)-g(y)|
&=|d(x,f(x))-d(y,f(y))| \
&=|d(x,f(x))-d(f(x),y)+d(f(x),y)-d(y,f(y))| \
&leq |d(x,f(x)-d(f(x),y)|+|d(f(x),y)-d(y,f(y))| \
&leq d(x,y)+d(f(x),f(y)) \
&leq 2d(x,y),
end{align*}

whre in the last step we used the hypothesis that $d(f(x), f(y))< d(x, y), forall x neq y ∈ X.$



b) To show continuity we take $delta= largefrac{varepsilon}{2}$. Then if $d(x,y)< delta$, we have by part a) that $|g(x)-g(y)|< varepsilon$.



To show that there exists $xin X$ such that $g(x)=0$, we follow AlexL's answer and note that since $g$ is continuous and $X$ is compact, $g$ attains its minimum at some $x in X$. Suppose that for that for $g(x)neq 0$, and so $f(x)neq x$. Then
$$g(f(x))=d(f(x),f(f(x)))<d(x,f(x))=g(x),$$
which is a contraction to $g$ attaining its minimum at $x$. It follows that $g(x)=0$, and this completes the proof.







real-analysis metric-spaces fixed-point-theorems complete-spaces






share|cite|improve this question















share|cite|improve this question













share|cite|improve this question




share|cite|improve this question








edited Dec 9 '18 at 7:48







Gaby Alfonso

















asked Oct 1 '18 at 0:03









Gaby AlfonsoGaby Alfonso

839316




839316








  • 1




    $begingroup$
    You have a small error. In the statement of a) an in the last displayed line of he proof of a) the inequality $<$ should be $leq$ to include the case $x=y.$ It would be easier to make this change than to re-write it to cover only the case $xne y$.
    $endgroup$
    – DanielWainfleet
    Nov 1 '18 at 7:47






  • 1




    $begingroup$
    Another way to prove that $g$ is continuous: It suffices that if $ lim_{nto infty}d(x,y_n)=0$ then $lim_{nto infty}|g(x)-g(y_n)|=0.$ Use your proof of a), writing $y_n$ for $y.$ Then $|g(x)-g(y_n)| leq 2d(x,y_n)$..... So if $d(x,y_n)to 0$ then $|g(x)-g(y_n)|to 0 .$
    $endgroup$
    – DanielWainfleet
    Nov 1 '18 at 7:59
















  • 1




    $begingroup$
    You have a small error. In the statement of a) an in the last displayed line of he proof of a) the inequality $<$ should be $leq$ to include the case $x=y.$ It would be easier to make this change than to re-write it to cover only the case $xne y$.
    $endgroup$
    – DanielWainfleet
    Nov 1 '18 at 7:47






  • 1




    $begingroup$
    Another way to prove that $g$ is continuous: It suffices that if $ lim_{nto infty}d(x,y_n)=0$ then $lim_{nto infty}|g(x)-g(y_n)|=0.$ Use your proof of a), writing $y_n$ for $y.$ Then $|g(x)-g(y_n)| leq 2d(x,y_n)$..... So if $d(x,y_n)to 0$ then $|g(x)-g(y_n)|to 0 .$
    $endgroup$
    – DanielWainfleet
    Nov 1 '18 at 7:59










1




1




$begingroup$
You have a small error. In the statement of a) an in the last displayed line of he proof of a) the inequality $<$ should be $leq$ to include the case $x=y.$ It would be easier to make this change than to re-write it to cover only the case $xne y$.
$endgroup$
– DanielWainfleet
Nov 1 '18 at 7:47




$begingroup$
You have a small error. In the statement of a) an in the last displayed line of he proof of a) the inequality $<$ should be $leq$ to include the case $x=y.$ It would be easier to make this change than to re-write it to cover only the case $xne y$.
$endgroup$
– DanielWainfleet
Nov 1 '18 at 7:47




1




1




$begingroup$
Another way to prove that $g$ is continuous: It suffices that if $ lim_{nto infty}d(x,y_n)=0$ then $lim_{nto infty}|g(x)-g(y_n)|=0.$ Use your proof of a), writing $y_n$ for $y.$ Then $|g(x)-g(y_n)| leq 2d(x,y_n)$..... So if $d(x,y_n)to 0$ then $|g(x)-g(y_n)|to 0 .$
$endgroup$
– DanielWainfleet
Nov 1 '18 at 7:59






$begingroup$
Another way to prove that $g$ is continuous: It suffices that if $ lim_{nto infty}d(x,y_n)=0$ then $lim_{nto infty}|g(x)-g(y_n)|=0.$ Use your proof of a), writing $y_n$ for $y.$ Then $|g(x)-g(y_n)| leq 2d(x,y_n)$..... So if $d(x,y_n)to 0$ then $|g(x)-g(y_n)|to 0 .$
$endgroup$
– DanielWainfleet
Nov 1 '18 at 7:59












1 Answer
1






active

oldest

votes


















1












$begingroup$

Since $X$ is compact and $g$ continuous, there exists $x_0 in X$ such that $g(x_0)$ is minimal. I let you show that we must have $g(x_0)=0$.






share|cite|improve this answer









$endgroup$













    Your Answer





    StackExchange.ifUsing("editor", function () {
    return StackExchange.using("mathjaxEditing", function () {
    StackExchange.MarkdownEditor.creationCallbacks.add(function (editor, postfix) {
    StackExchange.mathjaxEditing.prepareWmdForMathJax(editor, postfix, [["$", "$"], ["\\(","\\)"]]);
    });
    });
    }, "mathjax-editing");

    StackExchange.ready(function() {
    var channelOptions = {
    tags: "".split(" "),
    id: "69"
    };
    initTagRenderer("".split(" "), "".split(" "), channelOptions);

    StackExchange.using("externalEditor", function() {
    // Have to fire editor after snippets, if snippets enabled
    if (StackExchange.settings.snippets.snippetsEnabled) {
    StackExchange.using("snippets", function() {
    createEditor();
    });
    }
    else {
    createEditor();
    }
    });

    function createEditor() {
    StackExchange.prepareEditor({
    heartbeatType: 'answer',
    autoActivateHeartbeat: false,
    convertImagesToLinks: true,
    noModals: true,
    showLowRepImageUploadWarning: true,
    reputationToPostImages: 10,
    bindNavPrevention: true,
    postfix: "",
    imageUploader: {
    brandingHtml: "Powered by u003ca class="icon-imgur-white" href="https://imgur.com/"u003eu003c/au003e",
    contentPolicyHtml: "User contributions licensed under u003ca href="https://creativecommons.org/licenses/by-sa/3.0/"u003ecc by-sa 3.0 with attribution requiredu003c/au003e u003ca href="https://stackoverflow.com/legal/content-policy"u003e(content policy)u003c/au003e",
    allowUrls: true
    },
    noCode: true, onDemand: true,
    discardSelector: ".discard-answer"
    ,immediatelyShowMarkdownHelp:true
    });


    }
    });














    draft saved

    draft discarded


















    StackExchange.ready(
    function () {
    StackExchange.openid.initPostLogin('.new-post-login', 'https%3a%2f%2fmath.stackexchange.com%2fquestions%2f2937357%2ffixed-point-of-a-function-defined-in-terms-of-a-metric-on-a-compact-metric-space%23new-answer', 'question_page');
    }
    );

    Post as a guest















    Required, but never shown

























    1 Answer
    1






    active

    oldest

    votes








    1 Answer
    1






    active

    oldest

    votes









    active

    oldest

    votes






    active

    oldest

    votes









    1












    $begingroup$

    Since $X$ is compact and $g$ continuous, there exists $x_0 in X$ such that $g(x_0)$ is minimal. I let you show that we must have $g(x_0)=0$.






    share|cite|improve this answer









    $endgroup$


















      1












      $begingroup$

      Since $X$ is compact and $g$ continuous, there exists $x_0 in X$ such that $g(x_0)$ is minimal. I let you show that we must have $g(x_0)=0$.






      share|cite|improve this answer









      $endgroup$
















        1












        1








        1





        $begingroup$

        Since $X$ is compact and $g$ continuous, there exists $x_0 in X$ such that $g(x_0)$ is minimal. I let you show that we must have $g(x_0)=0$.






        share|cite|improve this answer









        $endgroup$



        Since $X$ is compact and $g$ continuous, there exists $x_0 in X$ such that $g(x_0)$ is minimal. I let you show that we must have $g(x_0)=0$.







        share|cite|improve this answer












        share|cite|improve this answer



        share|cite|improve this answer










        answered Oct 1 '18 at 0:11









        AlexLAlexL

        1,24416




        1,24416






























            draft saved

            draft discarded




















































            Thanks for contributing an answer to Mathematics Stack Exchange!


            • Please be sure to answer the question. Provide details and share your research!

            But avoid



            • Asking for help, clarification, or responding to other answers.

            • Making statements based on opinion; back them up with references or personal experience.


            Use MathJax to format equations. MathJax reference.


            To learn more, see our tips on writing great answers.




            draft saved


            draft discarded














            StackExchange.ready(
            function () {
            StackExchange.openid.initPostLogin('.new-post-login', 'https%3a%2f%2fmath.stackexchange.com%2fquestions%2f2937357%2ffixed-point-of-a-function-defined-in-terms-of-a-metric-on-a-compact-metric-space%23new-answer', 'question_page');
            }
            );

            Post as a guest















            Required, but never shown





















































            Required, but never shown














            Required, but never shown












            Required, but never shown







            Required, but never shown

































            Required, but never shown














            Required, but never shown












            Required, but never shown







            Required, but never shown







            Popular posts from this blog

            Le Mesnil-Réaume

            Ida-Boy-Ed-Garten

            web3.py web3.isConnected() returns false always